Question and Answers Forum

All Questions      Topic List

Relation and Functions Questions

Previous in All Question      Next in All Question      

Previous in Relation and Functions      Next in Relation and Functions      

Question Number 96211 by mathmax by abdo last updated on 30/May/20

solve inside C  (x−(1/x))^3  +(x−(1/x))^2  +(x−(1/x))+1 =0

$$\mathrm{solve}\:\mathrm{inside}\:\mathrm{C}\:\:\left(\mathrm{x}−\frac{\mathrm{1}}{\mathrm{x}}\right)^{\mathrm{3}} \:+\left(\mathrm{x}−\frac{\mathrm{1}}{\mathrm{x}}\right)^{\mathrm{2}} \:+\left(\mathrm{x}−\frac{\mathrm{1}}{\mathrm{x}}\right)+\mathrm{1}\:=\mathrm{0} \\ $$

Answered by mr W last updated on 30/May/20

let t=x−(1/x)  t^3 +t^2 +t+1=0  (t−1)(t^3 +t^2 +t+1)=0  ⇒t^4 −1=0  ⇒t^4 =1  ⇒t_k =cos ((2kπ)/4)+i sin ((2kπ)/4) with k=1,2,3  (k=0 is not root of original eqn.)  x−(1/x)=t  x^2 −tx−1=0  ⇒x=(t/2)±(√(((t/2))^2 +1))  ......

$${let}\:{t}={x}−\frac{\mathrm{1}}{{x}} \\ $$$${t}^{\mathrm{3}} +{t}^{\mathrm{2}} +{t}+\mathrm{1}=\mathrm{0} \\ $$$$\left({t}−\mathrm{1}\right)\left({t}^{\mathrm{3}} +{t}^{\mathrm{2}} +{t}+\mathrm{1}\right)=\mathrm{0} \\ $$$$\Rightarrow{t}^{\mathrm{4}} −\mathrm{1}=\mathrm{0} \\ $$$$\Rightarrow{t}^{\mathrm{4}} =\mathrm{1} \\ $$$$\Rightarrow{t}_{{k}} =\mathrm{cos}\:\frac{\mathrm{2}{k}\pi}{\mathrm{4}}+{i}\:\mathrm{sin}\:\frac{\mathrm{2}{k}\pi}{\mathrm{4}}\:{with}\:{k}=\mathrm{1},\mathrm{2},\mathrm{3} \\ $$$$\left({k}=\mathrm{0}\:{is}\:{not}\:{root}\:{of}\:{original}\:{eqn}.\right) \\ $$$${x}−\frac{\mathrm{1}}{{x}}={t} \\ $$$${x}^{\mathrm{2}} −{tx}−\mathrm{1}=\mathrm{0} \\ $$$$\Rightarrow{x}=\frac{{t}}{\mathrm{2}}\pm\sqrt{\left(\frac{{t}}{\mathrm{2}}\right)^{\mathrm{2}} +\mathrm{1}} \\ $$$$...... \\ $$

Answered by mathmax by abdo last updated on 30/May/20

let x−(1/x) =z  (e)⇒z^3  +z^2  +z +1 =0 ⇒((1−z^4 )/(1−z)) =0    (z≠1) ⇒  z^4  =1  and z ≠1  let z =r e^(iθ)  ⇒r^4  =1 and 4θ =2kπ ⇒r=1 and θ =((kπ)/2)  so the roots  are z_k =e^(i((kπ)/2))  and k ∈{1,2,3}  z_1 =e^((iπ)/2)  =i  ,z_2 =−1  ,z_3 =e^(i((3π)/2))  =e^(i(2π−(π/2)))  =−i  x−(1/x) =i ⇒x^2 −1 =ix ⇒x^2 −ix−1=0  Δ =−1+4 =3 ⇒x_1 =((i+(√3))/2)  x_2 =((i−(√3))/2)  x−(1/x) =−1 ⇒x^2 −1 =−x ⇒x^2 +x−1 =0  Δ =5 ⇒x_3 =((−1+(√5))/2) and x_4 =((−1−(√5))/2)  x−(1/x) =−i ⇒x^2 −1 =−ix ⇒x^2 +ix−1 =0  Δ =−1+4 =3 ⇒x_5 =((−i+(√3))/2) and x_6 =((−i−(√3))/2)  the roots of this equation are the complex  (x_i )_(1≤i≤6)

$$\mathrm{let}\:\mathrm{x}−\frac{\mathrm{1}}{\mathrm{x}}\:=\mathrm{z}\:\:\left(\mathrm{e}\right)\Rightarrow\mathrm{z}^{\mathrm{3}} \:+\mathrm{z}^{\mathrm{2}} \:+\mathrm{z}\:+\mathrm{1}\:=\mathrm{0}\:\Rightarrow\frac{\mathrm{1}−\mathrm{z}^{\mathrm{4}} }{\mathrm{1}−\mathrm{z}}\:=\mathrm{0}\:\:\:\:\left(\mathrm{z}\neq\mathrm{1}\right)\:\Rightarrow \\ $$$$\mathrm{z}^{\mathrm{4}} \:=\mathrm{1}\:\:\mathrm{and}\:\mathrm{z}\:\neq\mathrm{1}\:\:\mathrm{let}\:\mathrm{z}\:=\mathrm{r}\:\mathrm{e}^{\mathrm{i}\theta} \:\Rightarrow\mathrm{r}^{\mathrm{4}} \:=\mathrm{1}\:\mathrm{and}\:\mathrm{4}\theta\:=\mathrm{2k}\pi\:\Rightarrow\mathrm{r}=\mathrm{1}\:\mathrm{and}\:\theta\:=\frac{\mathrm{k}\pi}{\mathrm{2}} \\ $$$$\mathrm{so}\:\mathrm{the}\:\mathrm{roots}\:\:\mathrm{are}\:\mathrm{z}_{\mathrm{k}} =\mathrm{e}^{\mathrm{i}\frac{\mathrm{k}\pi}{\mathrm{2}}} \:\mathrm{and}\:\mathrm{k}\:\in\left\{\mathrm{1},\mathrm{2},\mathrm{3}\right\} \\ $$$$\mathrm{z}_{\mathrm{1}} =\mathrm{e}^{\frac{\mathrm{i}\pi}{\mathrm{2}}} \:=\mathrm{i}\:\:,\mathrm{z}_{\mathrm{2}} =−\mathrm{1}\:\:,\mathrm{z}_{\mathrm{3}} =\mathrm{e}^{\mathrm{i}\frac{\mathrm{3}\pi}{\mathrm{2}}} \:=\mathrm{e}^{\mathrm{i}\left(\mathrm{2}\pi−\frac{\pi}{\mathrm{2}}\right)} \:=−\mathrm{i} \\ $$$$\mathrm{x}−\frac{\mathrm{1}}{\mathrm{x}}\:=\mathrm{i}\:\Rightarrow\mathrm{x}^{\mathrm{2}} −\mathrm{1}\:=\mathrm{ix}\:\Rightarrow\mathrm{x}^{\mathrm{2}} −\mathrm{ix}−\mathrm{1}=\mathrm{0} \\ $$$$\Delta\:=−\mathrm{1}+\mathrm{4}\:=\mathrm{3}\:\Rightarrow\mathrm{x}_{\mathrm{1}} =\frac{\mathrm{i}+\sqrt{\mathrm{3}}}{\mathrm{2}}\:\:\mathrm{x}_{\mathrm{2}} =\frac{\mathrm{i}−\sqrt{\mathrm{3}}}{\mathrm{2}} \\ $$$$\mathrm{x}−\frac{\mathrm{1}}{\mathrm{x}}\:=−\mathrm{1}\:\Rightarrow\mathrm{x}^{\mathrm{2}} −\mathrm{1}\:=−\mathrm{x}\:\Rightarrow\mathrm{x}^{\mathrm{2}} +\mathrm{x}−\mathrm{1}\:=\mathrm{0} \\ $$$$\Delta\:=\mathrm{5}\:\Rightarrow\mathrm{x}_{\mathrm{3}} =\frac{−\mathrm{1}+\sqrt{\mathrm{5}}}{\mathrm{2}}\:\mathrm{and}\:\mathrm{x}_{\mathrm{4}} =\frac{−\mathrm{1}−\sqrt{\mathrm{5}}}{\mathrm{2}} \\ $$$$\mathrm{x}−\frac{\mathrm{1}}{\mathrm{x}}\:=−\mathrm{i}\:\Rightarrow\mathrm{x}^{\mathrm{2}} −\mathrm{1}\:=−\mathrm{ix}\:\Rightarrow\mathrm{x}^{\mathrm{2}} +\mathrm{ix}−\mathrm{1}\:=\mathrm{0} \\ $$$$\Delta\:=−\mathrm{1}+\mathrm{4}\:=\mathrm{3}\:\Rightarrow\mathrm{x}_{\mathrm{5}} =\frac{−\mathrm{i}+\sqrt{\mathrm{3}}}{\mathrm{2}}\:\mathrm{and}\:\mathrm{x}_{\mathrm{6}} =\frac{−\mathrm{i}−\sqrt{\mathrm{3}}}{\mathrm{2}} \\ $$$$\mathrm{the}\:\mathrm{roots}\:\mathrm{of}\:\mathrm{this}\:\mathrm{equation}\:\mathrm{are}\:\mathrm{the}\:\mathrm{complex}\:\:\left(\mathrm{x}_{\mathrm{i}} \right)_{\mathrm{1}\leqslant\mathrm{i}\leqslant\mathrm{6}} \\ $$

Answered by MJS last updated on 30/May/20

(x−(1/x))^3 +(x−(1/x))^2 +(x−(1/x))+1=0  ⇒  x^6 +x^5 −2x^4 −x^3 +2x^2 +x−1=0  (x^4 −x^2 +1)(x^2 +x−1)=0  and this is very easy to solve

$$\left({x}−\frac{\mathrm{1}}{{x}}\right)^{\mathrm{3}} +\left({x}−\frac{\mathrm{1}}{{x}}\right)^{\mathrm{2}} +\left({x}−\frac{\mathrm{1}}{{x}}\right)+\mathrm{1}=\mathrm{0} \\ $$$$\Rightarrow \\ $$$${x}^{\mathrm{6}} +{x}^{\mathrm{5}} −\mathrm{2}{x}^{\mathrm{4}} −{x}^{\mathrm{3}} +\mathrm{2}{x}^{\mathrm{2}} +{x}−\mathrm{1}=\mathrm{0} \\ $$$$\left({x}^{\mathrm{4}} −{x}^{\mathrm{2}} +\mathrm{1}\right)\left({x}^{\mathrm{2}} +{x}−\mathrm{1}\right)=\mathrm{0} \\ $$$$\mathrm{and}\:\mathrm{this}\:\mathrm{is}\:\mathrm{very}\:\mathrm{easy}\:\mathrm{to}\:\mathrm{solve} \\ $$

Commented by mr W last updated on 30/May/20

good idea!

$${good}\:{idea}! \\ $$

Terms of Service

Privacy Policy

Contact: info@tinkutara.com